Quiz Summary
0 of 228 Questions completed
Questions:
Information
You have already completed the quiz before. Hence you can not start it again.
Quiz is loading…
You must sign in or sign up to start the quiz.
You must first complete the following:
Results
Results
0 of 228 Questions answered correctly
Your time:
Time has elapsed
Categories
- Not categorized 0%
- 1
- 2
- 3
- 4
- 5
- 6
- 7
- 8
- 9
- 10
- 11
- 12
- 13
- 14
- 15
- 16
- 17
- 18
- 19
- 20
- 21
- 22
- 23
- 24
- 25
- 26
- 27
- 28
- 29
- 30
- 31
- 32
- 33
- 34
- 35
- 36
- 37
- 38
- 39
- 40
- 41
- 42
- 43
- 44
- 45
- 46
- 47
- 48
- 49
- 50
- 51
- 52
- 53
- 54
- 55
- 56
- 57
- 58
- 59
- 60
- 61
- 62
- 63
- 64
- 65
- 66
- 67
- 68
- 69
- 70
- 71
- 72
- 73
- 74
- 75
- 76
- 77
- 78
- 79
- 80
- 81
- 82
- 83
- 84
- 85
- 86
- 87
- 88
- 89
- 90
- 91
- 92
- 93
- 94
- 95
- 96
- 97
- 98
- 99
- 100
- 101
- 102
- 103
- 104
- 105
- 106
- 107
- 108
- 109
- 110
- 111
- 112
- 113
- 114
- 115
- 116
- 117
- 118
- 119
- 120
- 121
- 122
- 123
- 124
- 125
- 126
- 127
- 128
- 129
- 130
- 131
- 132
- 133
- 134
- 135
- 136
- 137
- 138
- 139
- 140
- 141
- 142
- 143
- 144
- 145
- 146
- 147
- 148
- 149
- 150
- 151
- 152
- 153
- 154
- 155
- 156
- 157
- 158
- 159
- 160
- 161
- 162
- 163
- 164
- 165
- 166
- 167
- 168
- 169
- 170
- 171
- 172
- 173
- 174
- 175
- 176
- 177
- 178
- 179
- 180
- 181
- 182
- 183
- 184
- 185
- 186
- 187
- 188
- 189
- 190
- 191
- 192
- 193
- 194
- 195
- 196
- 197
- 198
- 199
- 200
- 201
- 202
- 203
- 204
- 205
- 206
- 207
- 208
- 209
- 210
- 211
- 212
- 213
- 214
- 215
- 216
- 217
- 218
- 219
- 220
- 221
- 222
- 223
- 224
- 225
- 226
- 227
- 228
- Current
- Review
- Answered
- Correct
- Incorrect
-
Question 1 of 228
1. Question
1. BP-170/120 mmof hg with no end organ damage, what it called as?
CorrectIncorrect -
Question 2 of 228
2. Question
2. Identify the stage of hypertension.BP-152/80 mm of hg?
CorrectIncorrect -
Question 3 of 228
3. Question
3. ST elevation in lead II, III, AVF suggest? (MI= Myocardial Infarction)
CorrectIncorrect -
Question 4 of 228
4. Question
4. Young man who came to office immediately after 10 minutes of brisk walk with no previous history of hypertension. His blood pressure is 150/90mmof he? What is the next step?
CorrectIncorrect -
Question 5 of 228
5. Question
5. First symptoms in patient with Left heart failure is?
CorrectIncorrect -
Question 6 of 228
6. Question
6. The typical ECG changes associated with the first hour of acute pericarditis?
CorrectIncorrect -
Question 7 of 228
7. Question
7. Hyperkelemia is characterised by all of the following except?
CorrectIncorrect -
Question 8 of 228
8. Question
8. which of the following drug is cause of secondary hypertension?
CorrectIncorrect -
Question 9 of 228
9. Question
9. Hypertensive patient with sudden, severe chest and back pain has likely diagnosis of?
CorrectIncorrect -
Question 10 of 228
10. Question
10. Mild Hypertension with pregnancy first ling drug of choice ?
CorrectIncorrect -
Question 11 of 228
11. Question
11. Cardiac catheterization is performed to find out if you have disease of?
CorrectIncorrect -
Question 12 of 228
12. Question
12. in CABG (Coronary Artery Bypass Grafting) which artery is typically used as a pedicled graft to the left anterior descending coronary artery?
CorrectIncorrect -
Question 13 of 228
13. Question
13. Resistant hypertension when goal is not achieved.
CorrectIncorrect -
Question 14 of 228
14. Question
14. A 76 year old man presents with atrial fibrillation(AF) which is permanent. He is lucid and has specifically made an appointment with you to discuss the best way of stopping him “dying ” as a result of the AF.
Past medical historyHypertension (treated), Mild COPD.
Drug historyVerapamil (as treatment for AF),Amlodipine,Simvastatin,PRN inhalers
Social HistoryLives alone.Independent. Can carry out all ADL’s (activities of daily living)
What is the anticoagulation schedule you would recommend to prevent stroke?CorrectIncorrect -
Question 15 of 228
15. Question
15. A 50-year-old man with diabetes and hypertension complains of central chest pain while walking this morning. The pain lasted 10 minutes, was relieved by rest and had not occurred before. Examination and resting ECG are normal. Which is the SINGLE MOST appropriate next investigation?
CorrectIncorrect -
Question 16 of 228
16. Question
16. Radiofrequency ablation is done for?
CorrectIncorrect -
Question 17 of 228
17. Question
17. Most specific and sensitive test for myocardial damage is?
CorrectIncorrect -
Question 18 of 228
18. Question
18. The best test for establishing the diagnosis and the degree of myocardial dysfunction is
CorrectIncorrect -
Question 19 of 228
19. Question
19. Cardiac transplantation should not be performed in which of the following condition?
CorrectIncorrect -
Question 20 of 228
20. Question
20. Intraaortic Ballon Counter pulsation (IABP) is contraindicated in which of the following condition?
CorrectIncorrect -
Question 21 of 228
21. Question
21. Corrigan’s pulse, Quicke’spulse, water hammer pulse, traube’ssign, and Durozias’s sign are seen in??
CorrectIncorrect -
Question 22 of 228
22. Question
22. Charachteristic ECG changes in hypothermia is?
CorrectIncorrect -
Question 23 of 228
23. Question
23. 35 year’s woman presented with exertionaldyspnoea, precordial exam revealed loud S1 and rumbling mid condition murmur at apex. Possible complications of this condition can be all except?
CorrectIncorrect -
Question 24 of 228
24. Question
24. which of the following is a distinctive feature of left ventricular failure?
CorrectIncorrect -
Question 25 of 228
25. Question
25. the three stages of a one heart beat includes all of the following except?
CorrectIncorrect -
Question 26 of 228
26. Question
26. An 18-mo-old boy is brought to the paediatrician because of progressively worsening episodes of cyanosis. The child has moments where he turns blue and becomes dyspneic. During these episodes the child becomes irritable and remains in a squatting position. Physical examination reveals a small and thin child with clubbing of the fingers and toes. Lungs are normal. Heart auscultation reveals an RV lift and a grade III/VI harsh systolic ejection murmur at the upper left sternal border. Which of the following is the most likely diagnosis?
CorrectIncorrect -
Question 27 of 228
27. Question
27. Which of the following tests can assess risk or indicate prognosis for heart?
CorrectIncorrect -
Question 28 of 228
28. Question
28. Pulmonary Hypertension is Most Severe in which Untreated Valvular Disease?
CorrectIncorrect -
Question 29 of 228
29. Question
29. A 71 year old man had trans femoral catheterization for coronary angiography which revealed the presence of severe atherosclerotic narrowing of all major coronary arteries. Within hours of the procedure, he developed severe left lower quadrant abdominal pain, which was later associated with rebound tenderness, lividoreticularis (erythematous skin rash) of the abdominal skin, increasing hypertension, cold blue toes and progressively worsening renal failure. Laparotomy revealed necrosis of the descending colon. This was resected. Microscopic examination showed amorphous eosinophilic material containing slit-like spaces occluding arteries in the submucosa.
Which one of the following is the most likely explanation?CorrectIncorrect -
Question 30 of 228
30. Question
30. Which features are seen with the Tetrology of Fallot?
CorrectIncorrect -
Question 31 of 228
31. Question
31. Which of the following is most commonly associated with an increased risk for the development of torsades de pointes?
CorrectIncorrect -
Question 32 of 228
32. Question
32. A 65 y/o female is very weak with acute cholecystitis not responding to medical treatment. She has a history of myocardial infarction (MI) suffered about 5 months ago and currently has an irregular pulse and JVD. Her BP is normal. What’s the most appropriate next step?
CorrectIncorrect -
Question 33 of 228
33. Question
33. A 42yearold man with acute renal failure is confused. His serum potassium is 8.1 mEq/L . The most likely abnormal ECG finding is:
CorrectIncorrect -
Question 34 of 228
34. Question
34. A 45-year-old patient on hemodialysis for one week has noted that his blood pressure is more difficult to control. He reports good compliance with his medications, which include erythropoietin, ferrous sulfate, vancomycin, and vitamin D. His blood pressure is 180/99 mm Hg. Which of the following is the most likely cause for the worsening control of his blood pressure?
CorrectIncorrect -
Question 35 of 228
35. Question
35. Cardiovascular syncope what is true?
CorrectIncorrect -
Question 36 of 228
36. Question
36. Patient with hypertrophic subaortic stenosis referred from dentist before doing dental procedure what is true
CorrectIncorrect -
Question 37 of 228
37. Question
37. Patient presents to the emergency department with sudden onset of weakness in lower part of body with urinary incontinence since 4 hrs. CT scan shows ischemic stroke.Patient is already on aspirin. What would you do:
CorrectIncorrect -
Question 38 of 228
38. Question
38. 15 years old male patient complaining of joint pain & fever for 1 week , difficulty swallowing, liver 1 cm below costal and pancystolic murmur
CorrectIncorrect -
Question 39 of 228
39. Question
39. The best way of treating patient with BP= 130-139/80-85:
CorrectIncorrect -
Question 40 of 228
40. Question
40. A 70 y/o tall man fell off the curb while walking and is brought to the ER for severe chest pain. He is short of breath, pale, and perspiring heavily. P/E results: BP = 80/60 mmHg; HR = 120/min; JVD is (+); heart sounds are weak; bilateral breath sounds seem normal. He has a history of hypertension for the past 10 years. What’s the most likely Diagnosis?
CorrectIncorrect -
Question 41 of 228
41. Question
41. Male patient who is a known case of hypercholesterolemia, BMI: 31, his investigations show high total cholesterol, high LDL & high TG, of these investigations what is the most important risk factor for developing coronary artery disease?
CorrectIncorrect -
Question 42 of 228
42. Question
42. Adult with pulseless QRS different shapes:
CorrectIncorrect -
Question 43 of 228
43. Question
43. Patient was brought by his son. He was pulseless and ECG showed ventricular tachycardia, BP -70 systolic,Diastolic not measurable and patient is unconcious.what is your action?
CorrectIncorrect -
Question 44 of 228
44. Question
44. A 50-year-old woman is hospitalized for an acute inferior myocardial infarction for which she has undergone angioplasty to the right coronary artery. On day 4, she develops acute shortness of breath and hypotension. She does not have any chest pain. On examination, she is tachycardic with a 3/6 holosystolic murmur at the apex and bibasilar crackles. An electrocardiogram shows sinus tachycardia, and a chest x-ray shows pulmonary edema.
Which of the following is the most appropriate management for this patient’s problem?CorrectIncorrect -
Question 45 of 228
45. Question
45. One of the following is a characteristic of syncope (vasovagal attack):
CorrectIncorrect -
Question 46 of 228
46. Question
46. Patient who is a known case of posterior MI presented with syncope. Examination showed canon (a) wave with tachycardia, unreadable BP & wide QRS complexes on ECG. The diagnosis is:
CorrectIncorrect -
Question 47 of 228
47. Question
47. Which of the following indicate inferior wall Myocardial Infarciton in ECG?
CorrectIncorrect -
Question 48 of 228
48. Question
48. ECG shows ST elevation in the following leads V1, 2, 3, 4 & reciprocal changes in leads aVF & 2, what is the diagnosis?(MI=Myocardial infarction)
MI=Myocardial Infarction.CorrectIncorrect -
Question 49 of 228
49. Question
49. Arriving first to the scene, you find an unresponsive person with no pulse that has thrown up. You feel CPR is not something you are comfortable giving them. What would be the next best thing for you to do?
CorrectIncorrect -
Question 50 of 228
50. Question
50. Warfarin is given to all the following except:
CorrectIncorrect -
Question 51 of 228
51. Question
51. What will increase heart blood flow when increase load on heart?
CorrectIncorrect -
Question 52 of 228
52. Question
52. Most common cause of chronic hypertension:
CorrectIncorrect -
Question 53 of 228
53. Question
53. All are true about the best position in hearing the murmurs, EXCEPT:
CorrectIncorrect -
Question 54 of 228
54. Question
54. What is the most risk of antihypertensive drugs on elderly patient?
CorrectIncorrect -
Question 55 of 228
55. Question
55. Idiosyncratic hypotensive reaction to nitrates is reversed promptly by ?
CorrectIncorrect -
Question 56 of 228
56. Question
56. 60 year’s old male presented with history of 2 hours chest pain, ECG showed ST elevation on V1-V4 with multiple PVC (Premature Ventricular Contraction) & ventricular tachycardia. The management is:
CorrectIncorrect -
Question 57 of 228
57. Question
57. About ventricular fibrillation:
CorrectIncorrect -
Question 58 of 228
58. Question
58. Diastolic blowing murmur best heard in the left sternal border increased with squatting
CorrectIncorrect -
Question 59 of 228
59. Question
59. A 35 years old woman presented with exertional dyspnea. Precordial examination revealed loud S1 and rumbling mid diastolic murmur at apex. Possible complications of this condition can be all the following EXCEPT:
CorrectIncorrect -
Question 60 of 228
60. Question
60. A patient with sudden cardiac arrest, the ECG showed no electrical activities with oscillation of QRS with different shapes. The underlying process is
CorrectIncorrect -
Question 61 of 228
61. Question
61. Which of the following is the least likely to cause infective endocarditis:
CorrectIncorrect -
Question 62 of 228
62. Question
62. Sinus tachycardia and atrial flutter, how to differentiate?
CorrectIncorrect -
Question 63 of 228
63. Question
63. A patient has persistent >2mm ST elevation in V2-6 two hours following a myocardial infarction, with hypertension of 205/115 mmHg. He has already been given morphine and aspirin. What is the next management of choice?
CorrectIncorrect -
Question 64 of 228
64. Question
64. S3 occur in all of the following EXCEPT:
CorrectIncorrect -
Question 65 of 228
65. Question
65. Patient present with carotid artery obstruction by 80%, treatment of choice is?
CorrectIncorrect -
Question 66 of 228
66. Question
66. A patient is a known case of coronary artery disease, presents with its symptoms, to diagnose if that patient has MI or not, by first ECG & cardiac enzyme
CorrectIncorrect -
Question 67 of 228
67. Question
67. Treatment of chronic atrial fibrillation all, EXCEPT:
CorrectIncorrect -
Question 68 of 228
68. Question
68. Treatment of unstable angina include all EXCEPT:
CorrectIncorrect -
Question 69 of 228
69. Question
69. A patient is 5 weeks post-Myocardial Infarction, complaining of chest pain,fever and arthralagia:
CorrectIncorrect -
Question 70 of 228
70. Question
70. Old male come with CHF (Congestive Heart Failure) and pulmonary edema, what is the best initial therapy?
CorrectIncorrect -
Question 71 of 228
71. Question
71. Young patient came to ER with dyspnea and productive tinged blood frothy sputum, he is a known case of rheumatic heart disease, Atrial fibrillation and his cheeks have dusky rash, what is the diagnosis?
CorrectIncorrect -
Question 72 of 228
72. Question
72. Nitroglycerine causes all of the following, EXCEPT:
CorrectIncorrect -
Question 73 of 228
73. Question
73. Calcium channel blockers as nifedipine, verapamil and diltiazem are extremely useful in all of the following applications except?
CorrectIncorrect -
Question 74 of 228
74. Question
74. In atrial fibrillation and stroke, all are true , EXCEPT:
CorrectIncorrect -
Question 75 of 228
75. Question
75. Old man who had stable angina, all of the followings are correct except:
CorrectIncorrect -
Question 76 of 228
76. Question
76. Coarctation of the aorta is commonly associated with which of the following syndromes:
CorrectIncorrect -
Question 77 of 228
77. Question
77. 70 years old male was brought to the ER with sudden onset of pain in his left lower limb. The pain was severe with numbness. He had acute myocardial infarction 2 weeks ago and was discharged 24 hours prior to his presentation. The left leg was cold and pale, right leg was normal. The most likely diagnosis is:
CorrectIncorrect -
Question 78 of 228
78. Question
78. The antibiotic prophylaxis for endocarditis is:
CorrectIncorrect -
Question 79 of 228
79. Question
79. Patient with hypercholesterolemia, he should avoid:
CorrectIncorrect -
Question 80 of 228
80. Question
80. Difference between unstable and stable angina :
CorrectIncorrect -
Question 81 of 228
81. Question
81. Patient presented to ER with substernal chest pain. 3 months ago, the patient had complete physicalexamination, and was normal, ECG normal, only high LDL in which he started low fat diet and medication for it. What is the factor the doctor will take into considerations as a risk factor?
CorrectIncorrect -
Question 82 of 228
82. Question
82. A drug that is contraindicated in hypertrophic obstructive cardiomyopathy (HOCM):
CorrectIncorrect -
Question 83 of 228
83. Question
83. Fick method in determining cardiac output
CorrectIncorrect -
Question 84 of 228
84. Question
84. A man who has had MI you will follow the next enzyme
CorrectIncorrect -
Question 85 of 228
85. Question
85. Regarding murmur of mitral stenosis:
CorrectIncorrect -
Question 86 of 228
86. Question
86. What is the correct about unstable angina :
CorrectIncorrect -
Question 87 of 228
87. Question
87. Patient with history of AF + MI, what the best prevention for stroke is?
CorrectIncorrect -
Question 88 of 228
88. Question
88. Which is the most common condition associated with endocarditis?
CorrectIncorrect -
Question 89 of 228
89. Question
89. Drug used in treatment of CHF which decrease the mortality
CorrectIncorrect -
Question 90 of 228
90. Question
90. A patient is a known case of stable angina for 2 years, came c/o palpitation , Holtis monitor showed 1.2mm ST depression for 1 to 2 minutes in 5-10 minutes wt your Dx
CorrectIncorrect -
Question 91 of 228
91. Question
91. Furmingham risk score predict?
CorrectIncorrect -
Question 92 of 228
92. Question
92. What explains coronary artery disease the best?
CorrectIncorrect -
Question 93 of 228
93. Question
93. Which of the following is the most dangerous to a patient in terms of risk for CAD?
CorrectIncorrect -
Question 94 of 228
94. Question
94. A postmenopausal woman develops chest pain immediately on hearing the news of her son’s death in a war. Shedevelops acute chest pain, dyspnea, and ST segment elevation in leads V2 to V4 on electrocardiogram. Elevated levels of troponin confirm an acute myocardial infarction. Coronary angiography is normal including an absence of vasospasm on provocative testing. Echocardiography reveals apical left ventricular “ballooning.”
What is the presumed mechanism of this disorder?CorrectIncorrect -
Question 95 of 228
95. Question
95. Correcting which of the following risk factors for CAD(Coronary Artery Disease) will result in the most immediate benefit for the patient?
CorrectIncorrect -
Question 96 of 228
96. Question
96. A man with atypical chest pain is found to have normal nuclear isotope uptake in his myocardium at rest. On exercise, there is decreased uptake in the inferior wall. Two hours after exercise, the uptake of nuclear isotope returns to normal. What is the right thing to do?
CorrectIncorrect -
Question 97 of 228
97. Question
97. A 48-year-old woman comes to the office with chest pain that has been occurring over the last several weeks. The pain is not reliably related to exertion. She is comfortable now. The location of the pain is retrosternal. She has no hypertension, and the EKG is normal.
What is the most appropriate next step in management?CorrectIncorrect -
Question 98 of 228
98. Question
98. A 64-year-old man is placed on lisinopril as part of managing CAD in association with an ejection fraction of 24% and symptoms of breathlessness. Although he sometimes has rales on lung examination, the patient is asymptomatic today. Physical examination reveals minimal edema of the lower extremities. Blood tests reveal an elevated level of potassium that is present on a repeat measurement. EKG is unchanged.
How would you best manage the patient?CorrectIncorrect -
Question 99 of 228
99. Question
99. Which of the following is the most common adverse effect of statin medications?
CorrectIncorrect -
Question 100 of 228
100. Question
100. A 70-year-old woman comes to the emergency department with crushing substernal chest pain for the last hour. The pain radiates to her left arm and is associated with anxiety, diaphoresis, and nausea. She describes the pain as “sore”and “dull” and clenches her fist in front of her chest. She has a history of hypertension.
Which of the following is most likely to be found in this patient?CorrectIncorrect -
Question 101 of 228
101. Question
101. A 70-year-old woman comes to the emergency department with crushing substernal chest pain for the last hour.
Which of the following EKG findings would be associated with the worst prognosis?CorrectIncorrect -
Question 102 of 228
102. Question
102. A 70-year-old woman comes to the emergency department with crushing substernal chest pain for the last hour. An
EKG shows ST segment elevation in V2 to V4.
What is the most appropriate next step in the management of this patient?CorrectIncorrect -
Question 103 of 228
103. Question
103. A 70-year-old woman comes to the emergency department with crushing substernal chest pain for the last hour. An EKG shows ST segment elevation in V2 to V4. Aspirin has been given to the patient to chew.
What is the most appropriate next step in the management of this patient?CorrectIncorrect -
Question 104 of 228
104. Question
104. Which of the following is most important in decreasing the risk of restenosis of the coronary artery after PCI?
CorrectIncorrect -
Question 105 of 228
105. Question
105. A patient comes to a small rural hospital without a catheterization laboratory. The patient has chest pain and ST segment elevation. What is the most appropriate next step in the management of the patient?
CorrectIncorrect -
Question 106 of 228
106. Question
106. A man comes to the emergency department with chest pain for the last hour that is crushing in quality and does not change with respiration or the position of his body. An EKG shows ST segment depression in leads V2 to V4. Aspirin has been given.
What is the most appropriate next step in the management of this patient?CorrectIncorrect -
Question 107 of 228
107. Question
107. Which of the following is the most common cause of death from CHF?
CorrectIncorrect -
Question 108 of 228
108. Question
108. A 74-year-old African American man with a history of dilated cardiomyopathy secondary to MI in the past is seen in the office for routine evaluation. He is asymptomatic and is maintained on lisinopril, furosemide, metoprolol, aspirin, and digoxin. Lab tests reveal a persistently elevated potassium level. The EKG is
unchanged. What is the best management?CorrectIncorrect -
Question 109 of 228
109. Question
109. A 74-year-old woman comes to the emergency department with the acute onset of shortness of breath, respiratory rate of 38 per minute, rales to her apices, S3 gallop, and jugulovenous distension. What is the best initial step in the management of this patient?
CorrectIncorrect -
Question 110 of 228
110. Question
110. A 78-year-old man with a history of lung cancer comes to the emergency department with several days of increasing
shortness of breath. He became somewhat lightheaded today, and that is what has brought him to the hospital. On physical
examination, he has a blood pressure of 106/70 mm Hg; pulse of 112 bpm; jugulovenous distention; and the lungs are clear to
auscultation. The blood pressure drops to 92/58 mm Hg on inhalation.Which of the following is the most appropriate to confirm the
diagnosis?CorrectIncorrect -
Question 111 of 228
111. Question
111. A 67-year-old man comes to the emergency department with the
sudden onset of chest pain. He also has pain between his
scapulae. He has a history of hypertension and tobacco
smoking. His blood pressure is 169/108 mm Hg.
What is the best initial test?CorrectIncorrect -
Question 112 of 228
112. Question
112. A 67-year-old man comes to the emergency department with the
sudden onset of chest pain. He also has pain between his
scapulae. He has a history of hypertension and tobacco
smoking. His blood pressure is 169/108 mm Hg.
What is the most accurate test?CorrectIncorrect -
Question 113 of 228
113. Question
113. Which of the following is the most appropriate screening for
aortic aneurysm?CorrectIncorrect -
Question 114 of 228
114. Question
114. Which of the following is the most dangerous to a pregnant woman?
CorrectIncorrect -
Question 115 of 228
115. Question
1. BP-170/120 mmof hg with no end organ damage, what it called as?
CorrectIncorrect -
Question 116 of 228
116. Question
2. Identify the stage of hypertension.BP-152/80 mm of hg?
CorrectIncorrect -
Question 117 of 228
117. Question
3. ST elevation in lead II, III, AVF suggest? (MI= Myocardial Infarction)
CorrectIncorrect -
Question 118 of 228
118. Question
4. Young man who came to office immediately after 10 minutes of brisk walk with no previous history of hypertension. His blood pressure is 150/90mmof he? What is the next step?
CorrectIncorrect -
Question 119 of 228
119. Question
5. First symptoms in patient with Left heart failure is?
CorrectIncorrect -
Question 120 of 228
120. Question
6. The typical ECG changes associated with the first hour of acute pericarditis?
CorrectIncorrect -
Question 121 of 228
121. Question
7. Hyperkelemia is characterised by all of the following except?
CorrectIncorrect -
Question 122 of 228
122. Question
8. which of the following drug is cause of secondary hypertension?
CorrectIncorrect -
Question 123 of 228
123. Question
9. Hypertensive patient with sudden, severe chest and back pain has likely diagnosis of?
CorrectIncorrect -
Question 124 of 228
124. Question
10. Mild Hypertension with pregnancy first ling drug of choice ?
CorrectIncorrect -
Question 125 of 228
125. Question
11. Cardiac catheterization is performed to find out if you have disease of?
CorrectIncorrect -
Question 126 of 228
126. Question
12. in CABG (Coronary Artery Bypass Grafting) which artery is typically used as a pedicled graft to the left anterior descending coronary artery?
CorrectIncorrect -
Question 127 of 228
127. Question
13. Resistant hypertension when goal is not achieved.
CorrectIncorrect -
Question 128 of 228
128. Question
14. A 76 year old man presents with atrial fibrillation(AF) which is permanent. He is lucid and has specifically made an appointment with you to discuss the best way of stopping him “dying ” as a result of the AF.
Past medical historyHypertension (treated), Mild COPD.
Drug historyVerapamil (as treatment for AF),Amlodipine,Simvastatin,PRN inhalers
Social HistoryLives alone.Independent. Can carry out all ADL’s (activities of daily living)
What is the anticoagulation schedule you would recommend to prevent stroke?CorrectIncorrect -
Question 129 of 228
129. Question
15. A 50-year-old man with diabetes and hypertension complains of central chest pain while walking this morning. The pain lasted 10 minutes, was relieved by rest and had not occurred before. Examination and resting ECG are normal. Which is the SINGLE MOST appropriate next investigation?
CorrectIncorrect -
Question 130 of 228
130. Question
16. Radiofrequency ablation is done for?
CorrectIncorrect -
Question 131 of 228
131. Question
17. Most specific and sensitive test for myocardial damage is?
CorrectIncorrect -
Question 132 of 228
132. Question
18. The best test for establishing the diagnosis and the degree of myocardial dysfunction is
CorrectIncorrect -
Question 133 of 228
133. Question
19. Cardiac transplantation should not be performed in which of the following condition?
CorrectIncorrect -
Question 134 of 228
134. Question
20. Intraaortic Ballon Counter pulsation (IABP) is contraindicated in which of the following condition?
CorrectIncorrect -
Question 135 of 228
135. Question
21. Corrigan’s pulse, Quicke’spulse, water hammer pulse, traube’ssign, and Durozias’s sign are seen in??
CorrectIncorrect -
Question 136 of 228
136. Question
22. Charachteristic ECG changes in hypothermia is?
CorrectIncorrect -
Question 137 of 228
137. Question
23. 35 year’s woman presented with exertionaldyspnoea, precordial exam revealed loud S1 and rumbling mid condition murmur at apex. Possible complications of this condition can be all except?
CorrectIncorrect -
Question 138 of 228
138. Question
24. which of the following is a distinctive feature of left ventricular failure?
CorrectIncorrect -
Question 139 of 228
139. Question
25. the three stages of a one heart beat includes all of the following except?
CorrectIncorrect -
Question 140 of 228
140. Question
26. An 18-mo-old boy is brought to the paediatrician because of progressively worsening episodes of cyanosis. The child has moments where he turns blue and becomes dyspneic. During these episodes the child becomes irritable and remains in a squatting position. Physical examination reveals a small and thin child with clubbing of the fingers and toes. Lungs are normal. Heart auscultation reveals an RV lift and a grade III/VI harsh systolic ejection murmur at the upper left sternal border. Which of the following is the most likely diagnosis?
CorrectIncorrect -
Question 141 of 228
141. Question
27. Which of the following tests can assess risk or indicate prognosis for heart?
CorrectIncorrect -
Question 142 of 228
142. Question
28. Pulmonary Hypertension is Most Severe in which Untreated Valvular Disease?
CorrectIncorrect -
Question 143 of 228
143. Question
29. A 71 year old man had trans femoral catheterization for coronary angiography which revealed the presence of severe atherosclerotic narrowing of all major coronary arteries. Within hours of the procedure, he developed severe left lower quadrant abdominal pain, which was later associated with rebound tenderness, lividoreticularis (erythematous skin rash) of the abdominal skin, increasing hypertension, cold blue toes and progressively worsening renal failure. Laparotomy revealed necrosis of the descending colon. This was resected. Microscopic examination showed amorphous eosinophilic material containing slit-like spaces occluding arteries in the submucosa.
Which one of the following is the most likely explanation?CorrectIncorrect -
Question 144 of 228
144. Question
30. Which features are seen with the Tetrology of Fallot?
CorrectIncorrect -
Question 145 of 228
145. Question
31. Which of the following is most commonly associated with an increased risk for the development of torsades de pointes?
CorrectIncorrect -
Question 146 of 228
146. Question
32. A 65 y/o female is very weak with acute cholecystitis not responding to medical treatment. She has a history of myocardial infarction (MI) suffered about 5 months ago and currently has an irregular pulse and JVD. Her BP is normal. What’s the most appropriate next step?
CorrectIncorrect -
Question 147 of 228
147. Question
33. A 42yearold man with acute renal failure is confused. His serum potassium is 8.1 mEq/L . The most likely abnormal ECG finding is:
CorrectIncorrect -
Question 148 of 228
148. Question
34. A 45-year-old patient on hemodialysis for one week has noted that his blood pressure is more difficult to control. He reports good compliance with his medications, which include erythropoietin, ferrous sulfate, vancomycin, and vitamin D. His blood pressure is 180/99 mm Hg. Which of the following is the most likely cause for the worsening control of his blood pressure?
CorrectIncorrect -
Question 149 of 228
149. Question
35. Cardiovascular syncope what is true?
CorrectIncorrect -
Question 150 of 228
150. Question
36. Patient with hypertrophic subaortic stenosis referred from dentist before doing dental procedure what is true
CorrectIncorrect -
Question 151 of 228
151. Question
37. Patient presents to the emergency department with sudden onset of weakness in lower part of body with urinary incontinence since 4 hrs. CT scan shows ischemic stroke.Patient is already on aspirin. What would you do:
CorrectIncorrect -
Question 152 of 228
152. Question
38. 15 years old male patient complaining of joint pain & fever for 1 week , difficulty swallowing, liver 1 cm below costal and pancystolic murmur
CorrectIncorrect -
Question 153 of 228
153. Question
39. The best way of treating patient with BP= 130-139/80-85:
CorrectIncorrect -
Question 154 of 228
154. Question
40. A 70 y/o tall man fell off the curb while walking and is brought to the ER for severe chest pain. He is short of breath, pale, and perspiring heavily. P/E results: BP = 80/60 mmHg; HR = 120/min; JVD is (+); heart sounds are weak; bilateral breath sounds seem normal. He has a history of hypertension for the past 10 years. What’s the most likely Diagnosis?
CorrectIncorrect -
Question 155 of 228
155. Question
41. Male patient who is a known case of hypercholesterolemia, BMI: 31, his investigations show high total cholesterol, high LDL & high TG, of these investigations what is the most important risk factor for developing coronary artery disease?
CorrectIncorrect -
Question 156 of 228
156. Question
42. Adult with pulseless QRS different shapes:
CorrectIncorrect -
Question 157 of 228
157. Question
43. Patient was brought by his son. He was pulseless and ECG showed ventricular tachycardia, BP -70 systolic,Diastolic not measurable and patient is unconcious.what is your action?
CorrectIncorrect -
Question 158 of 228
158. Question
44. A 50-year-old woman is hospitalized for an acute inferior myocardial infarction for which she has undergone angioplasty to the right coronary artery. On day 4, she develops acute shortness of breath and hypotension. She does not have any chest pain. On examination, she is tachycardic with a 3/6 holosystolic murmur at the apex and bibasilar crackles. An electrocardiogram shows sinus tachycardia, and a chest x-ray shows pulmonary edema.
Which of the following is the most appropriate management for this patient’s problem?CorrectIncorrect -
Question 159 of 228
159. Question
45. One of the following is a characteristic of syncope (vasovagal attack):
CorrectIncorrect -
Question 160 of 228
160. Question
46. Patient who is a known case of posterior MI presented with syncope. Examination showed canon (a) wave with tachycardia, unreadable BP & wide QRS complexes on ECG. The diagnosis is:
CorrectIncorrect -
Question 161 of 228
161. Question
47. Which of the following indicate inferior wall Myocardial Infarciton in ECG?
CorrectIncorrect -
Question 162 of 228
162. Question
48. ECG shows ST elevation in the following leads V1, 2, 3, 4 & reciprocal changes in leads aVF & 2, what is the diagnosis?(MI=Myocardial infarction)
MI=Myocardial Infarction.CorrectIncorrect -
Question 163 of 228
163. Question
49. Arriving first to the scene, you find an unresponsive person with no pulse that has thrown up. You feel CPR is not something you are comfortable giving them. What would be the next best thing for you to do?
CorrectIncorrect -
Question 164 of 228
164. Question
50. Warfarin is given to all the following except:
CorrectIncorrect -
Question 165 of 228
165. Question
51. What will increase heart blood flow when increase load on heart?
CorrectIncorrect -
Question 166 of 228
166. Question
52. Most common cause of chronic hypertension:
CorrectIncorrect -
Question 167 of 228
167. Question
53. All are true about the best position in hearing the murmurs, EXCEPT:
CorrectIncorrect -
Question 168 of 228
168. Question
54. What is the most risk of antihypertensive drugs on elderly patient?
CorrectIncorrect -
Question 169 of 228
169. Question
55. Idiosyncratic hypotensive reaction to nitrates is reversed promptly by ?
CorrectIncorrect -
Question 170 of 228
170. Question
56. 60 year’s old male presented with history of 2 hours chest pain, ECG showed ST elevation on V1-V4 with multiple PVC (Premature Ventricular Contraction) & ventricular tachycardia. The management is:
CorrectIncorrect -
Question 171 of 228
171. Question
57. About ventricular fibrillation:
CorrectIncorrect -
Question 172 of 228
172. Question
58. Diastolic blowing murmur best heard in the left sternal border increased with squatting
CorrectIncorrect -
Question 173 of 228
173. Question
59. A 35 years old woman presented with exertional dyspnea. Precordial examination revealed loud S1 and rumbling mid diastolic murmur at apex. Possible complications of this condition can be all the following EXCEPT:
CorrectIncorrect -
Question 174 of 228
174. Question
60. A patient with sudden cardiac arrest, the ECG showed no electrical activities with oscillation of QRS with different shapes. The underlying process is
CorrectIncorrect -
Question 175 of 228
175. Question
61. Which of the following is the least likely to cause infective endocarditis:
CorrectIncorrect -
Question 176 of 228
176. Question
62. Sinus tachycardia and atrial flutter, how to differentiate?
CorrectIncorrect -
Question 177 of 228
177. Question
63. A patient has persistent >2mm ST elevation in V2-6 two hours following a myocardial infarction, with hypertension of 205/115 mmHg. He has already been given morphine and aspirin. What is the next management of choice?
CorrectIncorrect -
Question 178 of 228
178. Question
64. S3 occur in all of the following EXCEPT:
CorrectIncorrect -
Question 179 of 228
179. Question
65. Patient present with carotid artery obstruction by 80%, treatment of choice is?
CorrectIncorrect -
Question 180 of 228
180. Question
66. A patient is a known case of coronary artery disease, presents with its symptoms, to diagnose if that patient has MI or not, by first ECG & cardiac enzyme
CorrectIncorrect -
Question 181 of 228
181. Question
67. Treatment of chronic atrial fibrillation all, EXCEPT:
CorrectIncorrect -
Question 182 of 228
182. Question
68. Treatment of unstable angina include all EXCEPT:
CorrectIncorrect -
Question 183 of 228
183. Question
69. A patient is 5 weeks post-Myocardial Infarction, complaining of chest pain,fever and arthralagia:
CorrectIncorrect -
Question 184 of 228
184. Question
70. Old male come with CHF (Congestive Heart Failure) and pulmonary edema, what is the best initial therapy?
CorrectIncorrect -
Question 185 of 228
185. Question
71. Young patient came to ER with dyspnea and productive tinged blood frothy sputum, he is a known case of rheumatic heart disease, Atrial fibrillation and his cheeks have dusky rash, what is the diagnosis?
CorrectIncorrect -
Question 186 of 228
186. Question
72. Nitroglycerine causes all of the following, EXCEPT:
CorrectIncorrect -
Question 187 of 228
187. Question
73. Calcium channel blockers as nifedipine, verapamil and diltiazem are extremely useful in all of the following applications except?
CorrectIncorrect -
Question 188 of 228
188. Question
74. In atrial fibrillation and stroke, all are true , EXCEPT:
CorrectIncorrect -
Question 189 of 228
189. Question
75. Old man who had stable angina, all of the followings are correct except:
CorrectIncorrect -
Question 190 of 228
190. Question
76. Coarctation of the aorta is commonly associated with which of the following syndromes:
CorrectIncorrect -
Question 191 of 228
191. Question
77. 70 years old male was brought to the ER with sudden onset of pain in his left lower limb. The pain was severe with numbness. He had acute myocardial infarction 2 weeks ago and was discharged 24 hours prior to his presentation. The left leg was cold and pale, right leg was normal. The most likely diagnosis is:
CorrectIncorrect -
Question 192 of 228
192. Question
78. The antibiotic prophylaxis for endocarditis is:
CorrectIncorrect -
Question 193 of 228
193. Question
79. Patient with hypercholesterolemia, he should avoid:
CorrectIncorrect -
Question 194 of 228
194. Question
80. Difference between unstable and stable angina :
CorrectIncorrect -
Question 195 of 228
195. Question
81. Patient presented to ER with substernal chest pain. 3 months ago, the patient had complete physicalexamination, and was normal, ECG normal, only high LDL in which he started low fat diet and medication for it. What is the factor the doctor will take into considerations as a risk factor?
CorrectIncorrect -
Question 196 of 228
196. Question
82. A drug that is contraindicated in hypertrophic obstructive cardiomyopathy (HOCM):
CorrectIncorrect -
Question 197 of 228
197. Question
83. Fick method in determining cardiac output
CorrectIncorrect -
Question 198 of 228
198. Question
84. A man who has had MI you will follow the next enzyme
CorrectIncorrect -
Question 199 of 228
199. Question
85. Regarding murmur of mitral stenosis:
CorrectIncorrect -
Question 200 of 228
200. Question
86. What is the correct about unstable angina :
CorrectIncorrect -
Question 201 of 228
201. Question
87. Patient with history of AF + MI, what the best prevention for stroke is?
CorrectIncorrect -
Question 202 of 228
202. Question
88. Which is the most common condition associated with endocarditis?
CorrectIncorrect -
Question 203 of 228
203. Question
89. Drug used in treatment of CHF which decrease the mortality
CorrectIncorrect -
Question 204 of 228
204. Question
90. A patient is a known case of stable angina for 2 years, came c/o palpitation , Holtis monitor showed 1.2mm ST depression for 1 to 2 minutes in 5-10 minutes wt your Dx
CorrectIncorrect -
Question 205 of 228
205. Question
91. Furmingham risk score predict?
CorrectIncorrect -
Question 206 of 228
206. Question
92. What explains coronary artery disease the best?
CorrectIncorrect -
Question 207 of 228
207. Question
93. Which of the following is the most dangerous to a patient in terms of risk for CAD?
CorrectIncorrect -
Question 208 of 228
208. Question
94. A postmenopausal woman develops chest pain immediately on hearing the news of her son’s death in a war. Shedevelops acute chest pain, dyspnea, and ST segment elevation in leads V2 to V4 on electrocardiogram. Elevated levels of troponin confirm an acute myocardial infarction. Coronary angiography is normal including an absence of vasospasm on provocative testing. Echocardiography reveals apical left ventricular “ballooning.”
What is the presumed mechanism of this disorder?CorrectIncorrect -
Question 209 of 228
209. Question
95. Correcting which of the following risk factors for CAD(Coronary Artery Disease) will result in the most immediate benefit for the patient?
CorrectIncorrect -
Question 210 of 228
210. Question
96. A man with atypical chest pain is found to have normal nuclear isotope uptake in his myocardium at rest. On exercise, there is decreased uptake in the inferior wall. Two hours after exercise, the uptake of nuclear isotope returns to normal. What is the right thing to do?
CorrectIncorrect -
Question 211 of 228
211. Question
97. A 48-year-old woman comes to the office with chest pain that has been occurring over the last several weeks. The pain is not reliably related to exertion. She is comfortable now. The location of the pain is retrosternal. She has no hypertension, and the EKG is normal.
What is the most appropriate next step in management?CorrectIncorrect -
Question 212 of 228
212. Question
98. A 64-year-old man is placed on lisinopril as part of managing CAD in association with an ejection fraction of 24% and symptoms of breathlessness. Although he sometimes has rales on lung examination, the patient is asymptomatic today. Physical examination reveals minimal edema of the lower extremities. Blood tests reveal an elevated level of potassium that is present on a repeat measurement. EKG is unchanged.
How would you best manage the patient?CorrectIncorrect -
Question 213 of 228
213. Question
99. Which of the following is the most common adverse effect of statin medications?
CorrectIncorrect -
Question 214 of 228
214. Question
100. A 70-year-old woman comes to the emergency department with crushing substernal chest pain for the last hour. The pain radiates to her left arm and is associated with anxiety, diaphoresis, and nausea. She describes the pain as “sore”and “dull” and clenches her fist in front of her chest. She has a history of hypertension.
Which of the following is most likely to be found in this patient?CorrectIncorrect -
Question 215 of 228
215. Question
101. A 70-year-old woman comes to the emergency department with crushing substernal chest pain for the last hour.
Which of the following EKG findings would be associated with the worst prognosis?CorrectIncorrect -
Question 216 of 228
216. Question
102. A 70-year-old woman comes to the emergency department with crushing substernal chest pain for the last hour. An
EKG shows ST segment elevation in V2 to V4.
What is the most appropriate next step in the management of this patient?CorrectIncorrect -
Question 217 of 228
217. Question
103. A 70-year-old woman comes to the emergency department with crushing substernal chest pain for the last hour. An EKG shows ST segment elevation in V2 to V4. Aspirin has been given to the patient to chew.
What is the most appropriate next step in the management of this patient?CorrectIncorrect -
Question 218 of 228
218. Question
104. Which of the following is most important in decreasing the risk of restenosis of the coronary artery after PCI?
CorrectIncorrect -
Question 219 of 228
219. Question
105. A patient comes to a small rural hospital without a catheterization laboratory. The patient has chest pain and ST segment elevation. What is the most appropriate next step in the management of the patient?
CorrectIncorrect -
Question 220 of 228
220. Question
106. A man comes to the emergency department with chest pain for the last hour that is crushing in quality and does not change with respiration or the position of his body. An EKG shows ST segment depression in leads V2 to V4. Aspirin has been given.
What is the most appropriate next step in the management of this patient?CorrectIncorrect -
Question 221 of 228
221. Question
107. Which of the following is the most common cause of death from CHF?
CorrectIncorrect -
Question 222 of 228
222. Question
108. A 74-year-old African American man with a history of dilated cardiomyopathy secondary to MI in the past is seen in the office for routine evaluation. He is asymptomatic and is maintained on lisinopril, furosemide, metoprolol, aspirin, and digoxin. Lab tests reveal a persistently elevated potassium level. The EKG is
unchanged. What is the best management?CorrectIncorrect -
Question 223 of 228
223. Question
109. A 74-year-old woman comes to the emergency department with the acute onset of shortness of breath, respiratory rate of 38 per minute, rales to her apices, S3 gallop, and jugulovenous distension. What is the best initial step in the management of this patient?
CorrectIncorrect -
Question 224 of 228
224. Question
110. A 78-year-old man with a history of lung cancer comes to the emergency department with several days of increasing
shortness of breath. He became somewhat lightheaded today, and that is what has brought him to the hospital. On physical
examination, he has a blood pressure of 106/70 mm Hg; pulse of 112 bpm; jugulovenous distention; and the lungs are clear to
auscultation. The blood pressure drops to 92/58 mm Hg on inhalation.Which of the following is the most appropriate to confirm the
diagnosis?CorrectIncorrect -
Question 225 of 228
225. Question
111. A 67-year-old man comes to the emergency department with the
sudden onset of chest pain. He also has pain between his
scapulae. He has a history of hypertension and tobacco
smoking. His blood pressure is 169/108 mm Hg.
What is the best initial test?CorrectIncorrect -
Question 226 of 228
226. Question
112. A 67-year-old man comes to the emergency department with the
sudden onset of chest pain. He also has pain between his
scapulae. He has a history of hypertension and tobacco
smoking. His blood pressure is 169/108 mm Hg.
What is the most accurate test?CorrectIncorrect -
Question 227 of 228
227. Question
113. Which of the following is the most appropriate screening for
aortic aneurysm?CorrectIncorrect -
Question 228 of 228
228. Question
114. Which of the following is the most dangerous to a pregnant woman?
CorrectIncorrect